Divide using long division. Check your answer
(x3+3x2-x+2)/(x-1)

Answers

Answer 1

After performing long division on (x3+3x2-x+2)/(x-1) we get x² + 4x +3 leaving a remainder of 5.

Long division refers to the method of performing a division of two numbers or polynomials by hand. Furthermore, it involves several steps to evaluate in order to find a quotient and a remainder. It is considered an important and crucial form of practice in the branch of mathematics.                  

In the subject of dividing a polynomial, first, we divide the highest degree term of the dividend by the highest degree term of the divisor, and the remaining result is subtracted from the dividend. The calculation is as follows in the picture

To learn more about long division,

https://brainly.com/question/30059812

#SPJ4

                 

       


Related Questions

How is the volume of the first rectangular prism with base 2m by 3m and height 5m related to the volume of the second rectangular prism with base 3m by 5m and height 2​m? Explain.

Answers

Answer:

they have the same volume

Step-by-step explanation:

in order to find the volume of a rectangular prism you have to multiply the width,height and length.

2 × 3 × 5 = 30.since both prisms have the same numbers ,they have the same volume.

hope this helps :)

Answer:

The volumes of the two rectangular prisms are the same.

Step-by-step explanation:

For the first rectangular prism, the area of the base is 6 square meters, and the height is 5 meters, so the volume is 30 cubic meters.

For the second rectangular prism, the area of the base is 15 square meters, and the height is 2 meters, so the volume is 30 cubic meters, the same volume as the first rectangular prism.

Subtract the sum of -52 and - 638 from the sum of - 29 and 303 the value is

Answers

The solution of the expression is 964.

To start solving this problem, let's find the sum of -29 and 303. The sum of two numbers is the result when you add them together. So,

-29 + 303 = 274

The sum of -29 and 303 is 274.

Now, let's find the sum of -52 and -638. To add two negative numbers, you just add their absolute values and put a negative sign in front of the result. So,

-52 + (-638) = -690

The sum of -52 and -638 is -690.

Finally, the problem asks us to subtract the sum of -52 and -638 from the sum of -29 and 303. To subtract one sum from another, we just subtract the second sum from the first. So,

( -29 + 303 ) - ( -52 + -638 )

We can simplify the expression by rearranging the terms:

274 - (-690)

When we subtract a negative number, it's the same as adding its absolute value. So,

274 + 690 = 964

To know more about sum here

https://brainly.com/question/4196546

#SPJ4

In ΔHIJ, the measure of ∠J=90°, HI = 6. 7 feet, and JH = 4. 8 feet. Find the measure of ∠I to the nearest degree

Answers

The measure of angle I in the triangle HIJ using given measurements is equal to 46.05 degrees.

In the triangle HIJ,

The measure of angle J is equal to 90 degrees.

This implies ,

HI is the hypotenuse.

JH is the opposite side to angle I.

In triangle HIJ,

Using trigonometric ratio we get,

sin ∠I = Opposite side / Hypotenuse

Substitute the values we have,

⇒ sin ∠I = 4.8 / 6.7

⇒ sin ∠I = 0.72

Now , take sin⁻¹ both the side of the equation we get,

⇒sin⁻¹(sin ∠I) = sin⁻¹( 0.72 )

Here sin⁻¹(sin ∠I) = ∠I

⇒∠I = sin⁻¹( 0.72 )

⇒∠I = 46.05 degrees

Therefore, the measure of angle I is equal to 46.05 degrees.

learn more about measure here

brainly.com/question/21751552

#SPJ4

an x(bar) chart has a center line of 100, uses three-sigma control limits, and is based on a sample size of four. the process standard deviation is known to be six. if the process mean shifts from 100 to 92, what is the probability of detecting this shift on the first sample following the shift?

Answers

The probability of detecting this shift on the first sample following the shift would be 68.3%.

The probability of detecting can be calculated by taking the difference between the sample mean and the new process mean (92-100 = -8) and dividing it by the process standard deviation (6).

The result is -1.33, which is then used to calculate the probability of a sample mean falling within the three-sigma control limits.

The difference between the sample mean and the new process mean (92-100 = -8).

The difference by the process standard deviation (6).

The result is -1.33.

The number to calculate the probability of a sample mean falling within the three-sigma control limits.

The probability of detecting this shift on the first sample following the shift is 68.3%.For similar question on probability:

https://brainly.com/question/34187875

#SPJ11

Factor.
z squared+18z–19
(z - or + ?)(z- or + ?)

Answers

Answer:

(z - 1) (z + 19)

Step-by-step explanation:

z² + 18z - 19

Consider the form x² + bx + c. Find a pair of integers whose product is c and whose sum is b. In this case, whose product is −19 and whose sum is 18.

-1, 19

Write the factored form using these integers.

(z - 1) (z + 19)

In art class students are mixing blue and red paint to make purple paint. Hawa mixes 1 cup of blue paint and 5 cups of red paint. Jacob mixes 4 cups of blue paint and 13 cups of red paint. Use Hawa and Jacob’s percent of red paint to determine whose purple paint will be redder.

Hawa percent of red paint (to nearest whole number) =
%
Jacob percent of red paint (to nearest whole number) =
%
Hawa ’s purple paint will be redder.
Jacob’s purple paint will be redder.
The two purple paints will be equally red.

attempt 1 out of 2
Miracle Jackson
Which is more? (Percent Comparison)
Apr 10, 6:51:14 PM
Watch help video
In art class students are mixing blue and red paint to make purple paint. Hawa mixes 1 cup of blue paint and 5 cups of red paint. Jacob mixes 4 cups of blue paint and 13 cups of red paint. Use Hawa and Jacob’s percent of red paint to determine whose purple paint will be redder.

Answers

Answer:

Step-by-step explanation:

the profit p (in dollars) generated by selling x units of a certain commodity is given by the function p ( x ) = - 1500 + 12 x - 0.004 x ^ 2 What is the maximum profit, and how many units must be sold to generate it?

Answers

The profit (p) is $7500 generated by selling 1500 units of a certain commodity is given by the function p ( x ) = - 1500 + 12 x - 0.004 x²

To maximize our profit, we must locate the vertex of the parabola represented by this function. The x-value of the vertex indicates the number of units that must be sold to maximize profit.

We may use the formula for the x-coordinate of a parabola's vertex:

x = -b/2a

where a and b represent the coefficients of the quadratic function ax² + bx + c. In this situation, a = -0.004 and b = 12, resulting in:

x = -12 / 2(-0.004) = 1500

This indicates that when 1,500 units are sold, the profit is maximized.

To calculate the greatest profit, enter x = 1500 into the profit function:

P(1500) = -1500 + 12(1500) - 0.004(1500)^2

P(1500) = -1500 + 18000 - 9000

P(1500) = $7500

Therefore, the maximum possible profit is $7,500 and it is generated when 1,500 units are sold.

Learn more about Profit maximization:

https://brainly.com/question/30436087

#SPJ4

To achieve this maximum profit, exactly 1500 units must be sold.


To find the maximum profit and the number of units needed to generate it, we can use the given profit function p(x) = -1500 + 12x - 0.004x^2. We need to find the vertex of the parabola represented by this quadratic function, as the vertex will give us the maximum profit and the corresponding number of units.

Step 1: Identify the coefficients a, b, and c in the quadratic function.
In p(x) = -1500 + 12x - 0.004x^2, the coefficients are:
a = -0.004
b = 12
c = -1500

Step 2: Find the x-coordinate of the vertex using the formula x = -b / (2a).
x = -12 / (2 * -0.004) = -12 / -0.008 = 1500

Step 3: Find the maximum profit by substituting the x-coordinate into the profit function p(x).
p(1500) = -1500 + 12 * 1500 - 0.004 * 1500^2
p(1500) = -1500 + 18000 - 0.004 * 2250000
p(1500) = -1500 + 18000 - 9000
p(1500) = 7500

So, the maximum profit is $7,500, and 1,500 units must be sold to generate it.

To learn more about parabola: brainly.com/question/8227487

#SPJ11

the applet is selecting random samples from the town's population this year. what do we assume is true about this population of babies?

Answers

When the applet selects random samples from the town's population of babies, we assume that the population is large enough and diverse enough to accurately represent the characteristics and traits of the entire population.

We assume that the selection of the random samples is unbiased and that every member of the population has an equal chance of being selected for the sample.

Based on your question, we are discussing random samples taken from a town's population of babies this year. When selecting random samples from this population, we assume the following:

1. The population of babies is well-defined and includes all babies born in the town within the specified year.
2. The random samples are representative of the entire population, meaning that each baby has an equal chance of being selected in the sample.
3. The samples are independent, meaning that the selection of one baby does not influence the selection of another.

These assumptions ensure that the results obtained from the random samples can be generalized to the entire population of babies in the town for this year.

Learn more about random samples here:

https://brainly.com/question/15736806

#SPJ11

By assuming these conditions are met, we can perform statistical analyses on the random samples and make valid inferences about the entire population of babies in the town.

When an applet is selecting random samples from a town's population of babies this year, we typically assume the following about the population:
Independence:

Each baby selected in the sample is independent of the others, meaning that the outcome of one selection does not affect the outcome of another selection.
Randomness:

The applet chooses babies from the population in a random manner, ensuring that every baby has an equal chance of being selected.

Representativeness:

The random samples selected are representative of the entire population, meaning that the samples accurately reflect the characteristics of the town's population of babies as a whole.

For similar question on conditions.

https://brainly.com/question/28532738

#SPJ11

2. Find the area of the circle. Use 3.14 for r. Round to the nearest unit.
18 cm
01,017 cm²
0254 cm²
057 cm²
028 cm²
(1 point)

Answers

The area of the circle for given problem will be approx. 254 [tex]cm^2[/tex].

How to find the area of circle?

The formula for finding the area of a circle is given by:

Area =[tex]\pi* r^2[/tex]

where "π" (pi) is a mathematical constant approximately equal to 3.14159, and "r" represents the radius of the circle.

Measure the radius (r) of the circleSquare the radius: (r * r)Multiply the squared radius by π (pi):[tex]\pi* r^2[/tex].The result is the area of the circle.

Given,

Find the radius (r) of the circle. The radius is half of the diameter, so divide the diameter by 2:

Radius (r) = Diameter / 2 = 18 cm / 2 = 9 cm

Area = [tex]\pi * r^2[/tex] = [tex]3.14*(9 \;cm)^2[/tex] = [tex]254.34 \;cm^2[/tex] (rounded to two decimal places)

Learn more about Area of Circle here:

https://brainly.com/question/28642423

#SPJ1

Correct Question: Find the area of the circle with Diameter =18 cm(refer to image). Use 3.14 for π.( Round to the nearest unit).

How does the structure of each poem help to convey different feelings

Answers

The fraction of the total vote he should expect to get is 23/75.

What are fractions?

A fraction is a mathematical unit used to represent a portion of a whole or a ratio of two integers. They are shown as the top number, or numerator, and the bottom number, or denominator, separated by a line. The denominator is the total number of pieces that make up the whole, whereas the numerator is the number of shares or parts.

For instance, you may write 3/8 for the portion of pizza you consumed if there were 8 pieces and you only ate 3. This indicates that you consumed three of the pizza's eight equally sized portions.

From the given table we can determine the total number of votes are:

13 + 9 + 10 + 8 + 10 + 14 + 5 + 6 = 75.

Now, Jamal received 13 + 10 = 23 votes.

Thus, the fraction of the total vote he should expect to get is 23/75.

Learn more about fraction here:

https://brainly.com/question/10354322

#SPJ1

sally works in hr at a company with 20 full-time employees in one location. what employee census data should sally gather to prepare for a benefits bid? race, gender, and ethnicityage, marital status, and number of childrendisability and veteran statustenure and education levels

Answers

To prepare for a benefits bid, Sally should census gather data on the age, marital status, and number of children of the 20 full-time employees.

This information will help Sally determine what types of benefits would be most appealing to the employees and what types of benefits might be necessary to attract and retain talent.

Additionally, Sally should gather data on the tenure and education levels of the employees to help her understand what types of benefits might be necessary to incentivize employees to stay with the company long-term and to attract highly educated candidates.

Finally, Sally should consider gathering data on disability and veteran status to ensure that the company is providing adequate support for those employees who may require additional assistance.

Learn more about census data collection at

https://brainly.com/question/30627450

#SPJ4

Solve for x. -7.6 -1.2 + X 0.5​

Answers

To solve for x, we need to simplify the expression first by adding the numbers on the left side and right side of the equation:

-7.6 - 1.2 + x = 0.5

Adding -7.6 and -1.2, we get:

-8.8 + x = 0.5

Now we can isolate x by adding 8.8 to both sides of the equation:

-8.8 + x + 8.8 = 0.5 + 8.8

Simplifying, we get:

x = 9.3

Therefore, the solution for x in the equation -7.6 - 1.2 + x = 0.5 is x = 9.3.

The recipe for one batch of
chocolate chip cookies calls
for 11/3 cups of chocolate
chips. Luca made 3½ batches
of cookies. How many cups of
chocolate chips did he use?

Answers

Answer:

Step-by-step explanation:

1 + 1 divided by 3 x 3.5 = 4.66 or 4 2/3

Steven read that the actual distance between Memphis and Chicago is about 525 miles. On a map, the distance between the two cities is about 10 inches. Which is most likely the scale used to make the map?​

Answers

Answer:About 1,012 Inches

Step-by-step explanation:

a survey researcher is most likely to use a(n) question when the dimensions of the variables are well defined.

Answers

In contrast, open-ended questions allow for more in-depth and nuanced responses but can be more difficult to analyze due to the variability of responses.

A survey researcher is most likely to use a closed-ended question when the dimensions of the variables are well defined. Closed-ended questions provide respondents with a list of predefined response options to choose from, which allows for easier categorization and analysis of the data. This type of question is ideal for measuring specific and clearly defined variables, such as demographic information or attitudes towards a particular issue. In contrast, open-ended questions allow for more in-depth and nuanced responses but can be more difficult to analyze due to the variability of responses.

learn more about the dimensions of the variables

https://brainly.com/question/14553712

#SPJ11

an imaginary circle that goes through both retinae and the fixation point is known as

Answers

The Vieth-Müller Circle is an imaginary circle that passes between both retinae and the fixation point.

The Vieth-Müller Circle is an ophthalmology concept that depicts an imaginary circle that passes across the foveas (the primary points of the retinae that are responsible for acute, detailed vision) and the fixation point (the point at which the eyes are directed).

The Vieth-Müller Circle is significant because it helps to explain the phenomenon of binocular vision, which is the ability to perceive depth and three-dimensional space using both eyes together. The circle aids in the definition of the equivalent locations on the two retinae, which are sites that receive visual field information and are critical in combining the images from the two eyes into a single, three-dimensional perception.

To know more about retinae and fixation point, visit,

https://brainly.com/question/27123882

#SPJ4

The imaginary circle that passes through both retinae and the fixation point is called the horopter. The horopter is important in visual perception as it represents the set of points in space that stimulate corresponding points on each retina, which is necessary for binocular vision and depth perception.


The Horopter is an imaginary circle that passes through both retinae and the fixation point. In this context:

1. "Imaginary" refers to the fact that the Horopter is a theoretical concept rather than a physical object.
2. "Retinae" are the light-sensitive layers at the back of both eyes, which play a crucial role in processing visual information.
3. "Fixation" is the point where both eyes are focused on a single object in the visual field.

In brief, the Horopter represents a collection of points in the 3D space that are perceived as having the same depth or distance as the fixation point. It helps in understanding binocular vision and depth perception, as points on the Horopter contribute to forming a single, fused image from both eyes.

To learn more about depth perception : brainly.com/question/2372629

#SPJ11

The Olympic record for the men's 50-meter freestyle is 21.91 seconds. Express this speed in meters per second

Answers

Answer:

50 meters/21.91 seconds = 2.282 m/sec

PLEASE HELP DUE TODAY

Answers

Answer:

y=-1/12x+61/12

Step-by-step explanation:

y=-1/12x+61/12

Find the area of the shaded region

Answers

The area of the shaded region which is a semi-circle is 157ft.

What is semi-circle?

In geometry, a semicircle can be defined as a half of a circle formed by cutting the circle into two halves. It is formed when a line or cut passes through the center and touches the two end positions of the circle and the line is called the diameter of the circle.

The shaded region is a semi- circle.

The radius is 10 ft.

The formula for the area of semi- circle is π×r²/2 where π= 3.14 and r is the radius.

So, the area of the semi- circle is {3.14× (10)²}/ 2

                                                      = 157 ft.

Hence, the area of the shaded region which is a semi-circle is 157ft.

To know more about semi-circle check the below link:

https://brainly.com/question/15822332

#SPJ1

help me please like right now as soon as possible write the answer in terms of pi and round the answer to the nearest hundredths place I will give branliest

Answers

Thus, the total surface area of cylinder is found to be 480π sq. cm.

Explain about the surface area of cylinder:

A cylinder's surface area is made up of its two congruent, parallel circular sides added together with its curved surface area. You must determine the Base Area (B) and Curved Surface Area in order to determine the surface area of a cylinder (CSA).

As a result, the base area multiplied by two and the area of a curved surface add up to the surface area or total surface of a cylinder.

Given data:

radius r = 8 cm

Height h = 22 cm

Total surface area of cylinder = 2*area of circle + area of curved cylinder

TSA = 2πr² + 2πrh

TSA = 2π(8)² + 2π(8)(22)

TSA = 2π(64) + 2π(176)

TSA = 128π + 352π

TSA = 480π sq. cm.

Thus, the total surface area of cylinder is found to be 480π sq. cm.

Know more about the surface area of cylinder:

https://brainly.com/question/27440983

#SPJ1

Complete question-

Find the surface area of the cylinder with radius of 8 cm and height of 22 cm. write the answer in terms of pi and round the answer to the nearest hundredths place.

Phil is baking a pie with cranberries and apples. Apples cost $0.60/cup and cranberries cost $0.40/cup. Phil wants to spend no more than $4.20 on the fruit for his pie.

Question
What is an inequality that represents the combinations he can use?

Answers

Let a be the number of cups of apples and c be the number of cups of cranberries. An inequality that represents the combinations Phil can use is:

0.6a + 0.4c <= 4.2

This inequality ensures that the cost of the fruit used for the pie is no more than $4.20.

cara computes the mean and variance for the set 87, 46, 90, 78, and 89. she finds the mean to be 78. her steps for finding the variance are shown below. what is the first error cara made in computing the variance?

Answers

If Cara's calculation of the sum of the squared differences i.e. 1370 from the mean is incorrect, that would be the first error she made in computing the variance.

As the steps for finding the variance are not provided, it is difficult to determine the first error Cara made.

However, the formula for calculating the variance is:

Variance = (sum of the squared differences from the mean) / (number of observations)

The first error Cara may have made is in calculating the sum of the squared differences from the mean.

The correct steps to find the variance are:

Find the mean:

Mean = (87 + 46 + 90 + 78 + 89) / 5 = 78

Calculate the differences from the mean for each observation:

87 - 78 = 9

46 - 78 = -32

90 - 78 = 12

78 - 78 = 0

89 - 78 = 11

Square each difference:

[tex]9^2 = 81[/tex]

[tex](-32)^2 = 1024[/tex]

[tex]12^2 = 144[/tex]

[tex]0^2 = 0[/tex]

[tex]11^2 = 121[/tex]

Find the sum of the squared differences:

81 + 1024 + 144 + 0 + 121 = 1370

Divide the sum of squared differences by the number of observations:

Variance = 1370 / 5 = 274.

For similar question on variance.

https://brainly.com/question/15858152

#SPJ11

2. Which sequence of transformations takes the graph of y = k(x) to the graph of
y=-k(x + 1)?
A. Translate 1 to the right, reflect over the x-axis, then scale vertically by a factor of 1/2
B. Translate 1 to the left, scale vertically by 1/2 , then reflect over the y-axis.
C. Translate left by 1/2, then translate up 1.
D. Scale vertically by 1/2, reflect over the x-axis, then translate up 1.

Answers

The correct answer is option B. Translate 1 to the left, scale vertically by 1/2, then reflect over the y-axis.

What does term "transformation of a graph" means?

The process of modifying the shape, location, or features of a graph is often referred to as graph transformation. Graphs are visual representations of mathematical functions or data point connections, often represented on a coordinate plane.

Translations, reflections, rotations, dilations, and other changes to the look of a graph are examples of graph transformations.

For the given problem, Transformation to get the desired result can be carried out as:

Translate '1' to the left: The transformation "x + 1" in "-k(x + 1)" shifts the graph horizontally to the left by 1 unit.Scale vertically by '1/2' : The 1/2 factor in "-k(x + 1)" vertically scales the graph, compressing it vertically.Reflect over the y-axis: The minus sign before "k" in "-k(x + 1)" reflects the graph over the y-axis, flipping it horizontally.

Hence, to convert the graph of "y = k(x)" to the graph of "y = -k(x + 1)," the correct sequence of transformations is to translate 1 unit to the left, scale vertically by 1/2, and then reflect across the y-axis, which is option B.

Learn more about Graph Transformation here:

https://brainly.com/question/10059147

#SPJ1

Use the functions f(x)=√x+1, g(x)=2x-5, and h(x) = 3x² - 3 to complete the table.
x
4
10
20
34
52
f(g(x))

Answers

Answer:

To find the values of f(g(x)) for the given values of x, we need to first evaluate g(x) for each value of x, and then plug the result into f(x).

Using the given functions:

g(x) = 2x - 5

f(x) = √(x+1)

Therefore, we have:

f(g(x)) = √(g(x) + 1) = √(2x - 5 + 1) = √(2x - 4) = 2√(x - 2)

So, we can complete the table as follows:

x f(g(x))

4 2

10 4

20 6

34 8

52 10

Therefore, the completed table is:

x f(g(x))

4 2

10 4

20 6

34 8

52 10

Given this snippet of code, what is the value of x after executing the last statement? int x = 10, *y; y = &x; y = y + 1; *y = 100;

Answers

The value of x after executing the last statement is still 10.

After executing the last statement, the value of x is still 10. The snippet of code declares an integer variable x and a pointer variable y that points to the address of x. Then, y is incremented by 1 (which means it now points to the next memory location after x). Finally, the value 100 is assigned to the memory location pointed to by y, which is actually beyond the memory allocated for variable x. This can lead to unexpected behavior, but since the value of x is never modified directly, its value remains unchanged at 10.

Learn more about snippet of code here: brainly.com/question/28650328

#SPJ11

Please answer the question in the pdf. I just need the values for A, B, and C. I am offering 15 points. Thanks.

Answers

Recall the equation provided in the pdf:

   (125x ^ 3 * y ^ - 12) ^ (- 2/3) = (y ^ [A])/([B] * x ^ [c])

find A B and C.

The answer will be:

A = 8/3B = 3/4C = 8/3

Checkout the calculation of the exponential

We can solve this problem using the rules of exponents and algebraic manipulation.

Starting with the left-hand side of the equation:

(125x^3 * y^-12)^(-2/3)

Using the rule that (a * b)^c = a^c * b^c, we can rewrite the expression as:

125^(-2/3) * x^(-2) * y^(8)

Simplifying further, we can use the fact that a^(-n) = 1/(a^n) to get:

1/(5^2 * x^2 * y^8/3)

Now, we can see that the denominator on the right-hand side of the equation must be 5^2 * x^2 * y^8/3. To find the numerator, we need to simplify the expression y^A. Comparing exponents, we see that:

y^A = y^(8/3)

Therefore, we need to find a value of A such that A = 8/3. Solving for A, we get:

A = 8/3

Now, we can write the equation as:

y^(8/3)/(5^2 * x^2 * y^8/3) = y^(8/3)/(25 * x^2 * y^(8/3))

Comparing exponents again, we see that we need to find values of B and C such that:

B * C = 2

and

-8/3 = -C

Solving for C, we get:

C = 8/3

Substituting this value of C into the first equation, we get:

B * 8/3 = 2

Solving for B, we get:

B = 3/4

Therefore, the solution is:

A = 8/3

B = 3/4

C = 8/3

Learn more about exponential algebra here:

https://brainly.com/question/12940982

#SPJ1

tim wants his mean quiz score to be 90. his first 3 quiz scores were 86, 92, and 94. what score should he make on the 4th quiz in order to have a mean quiz score of exactly 90?

Answers

The score to be made on the 4th quiz in order to have a mean quiz score of exactly 90 is equal to 88.

Let us consider the score that Tim needs to get on his fourth quiz be x.

Score he needs to get in order to have a mean quiz score of 90,

Set up an equation using the formula for the mean ,

(mean score) = (sum of scores) / (number of scores)

If Tim wants his mean quiz score to be 90, then we have,

⇒ 90 = (86 + 92 + 94 + x) / 4

Multiplying both sides by 4, we get,

⇒360 = 86 + 92 + 94 + x

Simplifying this equation, we get,

⇒ x = 360 - 272

⇒ x = 88

Therefore, Tim needs to get a score of 88 on his fourth quiz in order to have a mean quiz score of exactly 90.

Learn more about score here

brainly.com/question/31177688

#SPJ4

First person to help me on this will be rewarded plsss

Answers

The compound interest investment earns more interest than the simple interest investment, and the difference becomes more significant over time.

After 10 years, the difference is relatively small, but after 20 years, the compound interest investment has earned over $3,700 more than the simple interest investment.

What are the amounts after 10 and 20 years of the investment?

To calculate the amount after 10 and 20 years, we can use the following formulas:

Simple Interest: A = P * (1 + r * t)

Compound Interest: A = P * (1 + r/n)^(n*t)

Where:

A is the amount after t yearsP is the principal amount (the initial investment)r is the annual interest rate (as a decimal)n is the number of times the interest is compounded per yeart is the time in years

Using these formulas and the information given in the table, we can calculate the amount after 10 years and 20 years for each type of investment.

For simple interest, the interest is not compounded, so we just add the interest to the principal:

Amount after 10 years: A = $50,000 * (1 + 0.0275 * 10) = $68,750

Amount after 20 years: A = $50,000 * (1 + 0.0275 * 20) = $87,500

For compound interest, we need to use the formula above with n=12 (since the interest is compounded monthly):

Amount after 10 years: A = $50,000 * (1 + 0.0275/12)^(12*10) = $68,704.23

Amount after 20 years: A = $50,000 * (1 + 0.0275/12)^(12*20) = $91,262.47

To compare the two investments, we can look at the difference between the amounts after 10 and 20 years:

Difference after 10 years: $68,750 - $68,704.23 = $45.77

Difference after 20 years: $87,500 - $91,262.47 = $3,762.47

Learn more about simple and compound interest at: https://brainly.com/question/2277782

#SPJ1

A spinner with repeated colors numbered from 1 to 8 is shown. Sections 1 and 8 are purple. Sections 2 and 3 are yellow. Sections 4, 5, and 6 are blue. Section 7 is red. Spinner divided evenly into eight sections with three colored blue, one red, two purple, and two yellow. Determine the theoretical probability of the spinner not landing on red, P. 0.125 0.250 0.675 0.875

Answers

The theoretical probability of the spinner not landing on red is 0.875.

How to determine the theoretical probability of the spinner not landing on red

The total number of sections on the spinner is 8, out of which only one section is red. Therefore, the probability of the spinner landing on red is:

P(Red) = 1/8

The probability of the spinner not landing on red would be the probability of landing on any other section, which is:

P(Not Red) = 1 - P(Red) = 1 - 1/8 = 7/8

Therefore, the theoretical probability of the spinner not landing on red is 7/8 or 0.875 in decimal form.

So, the correct answer is: 0.875.

Learn more about probability at https://brainly.com/question/24756209

#SPJ1

Answer:

D

Step-by-step explanation:

your laundry basket contains 4 plain socks: a red one, a blue one, a yellow one and a green one. the basket also contains 4 striped socks: a red striped one, a blue striped one, a yellow stripped one and a green stripped one. if you want to wear a plain sock on your left foot and a stripped sock on your right foot, how many options do you have?

Answers

The total number of probability options will be 8 options.

1. Red plain and Red striped.

2. Blue plain and Blue striped.

3. Yellow plain and Yellow striped.

4. Green plain and Green striped.

5. Red plain and Blue striped.

6. Blue plain and Yellow striped.

7. Yellow plain and Green striped.

8. Green plain and Red striped.

There are 8 socks in the basket, and you can choose 1 plain sock and 1 striped sock.

This means that there are 8 possible combinations of socks you could choose.

Count the number of socks in the basket. There are 8 total socks (4 plain and 4 striped).

The number of options: There are 8 possible combinations of socks you could choose, so you have 8 options for wearing a plain sock on your left foot and a striped sock on your right foot.

Therefore,

You have 8 options for wearing a plain sock on your left foot and a striped sock on your right foot.

For similar question on probability:

brainly.com/question/30034780

#SPJ11

Other Questions
. what is the present (year 0) value of the cash flow stream if the opportunity cost rate is 10 percent? The nurse is caring for a new mother who has a chlamydial infection. For which complications should the nurse assess the client's neonate? Select all that apply.PneumoniaPreterm birthMicrocephalyConjunctivitisCongenital cataracts First day on the job quiz what is the central idea of the statement Last Friday, AT&T closed at $41.68. AT&T pays an annual dividend of $1.98. Calculate the dividend yield help please! state the key features for the graph Eight years ago Zack& Co. had purchased an equipment fo: $1,200,000. This equipment was being depreciated on a straight line basis over a 12 year period to a 300,000 salvage value. The equipment has six more years of economic life. During this period the annual revenues and operating costs assocaited with this machine are expected to be $388,000 and $87,500, respectively. Zack is now considering replacing this machine with a more modern one. The old equipment can now be sold for $180,000. Investment in net working capital is expected to increase by $132,000 as a result of the investment. The new machine will cost $1,500,000 and another $120,000 will be needed to modify it. This machine falls into the ACRS 5-year class life. It. is also expected to have an economic life of SiX years. The annual revenue and operating costs from the new machine are expected to be $750,000 and $58,000, respectively. At the sixth year Zack expects to sell the nes machine for $180,000. Zack's marginal tax rate is 34%. The equipment will be equity financed. (14 pts.) 1 Please calculate Zack's Net Investment and the Net Cash flows for the next six years if the replacement decision is made. five rectangles, and have integer side lengths. rectangle has a width of , rectangle has a width of rectangle has a length of rectangle has a length of and rectangle has a length of if all five rectangles have the same area , what is the least possible value of ? Cules son los pacientes fciles en trabajo sobre los alcohlicos A stock recently paid a $5/share dividend and they currently have a constant growth policy with g=10%/year. They will maintain this policy for the next 3 years. The growth rate will fall down to 3%/year after year 3 into perpetuity (i.e., from year 3 to year 4, the growth rate is 3%). R = 15%. Calculate the stock price. A stock will pay the following dividends over the next 5 years: $4 in year 1, $4.5 in year 2, $5 in year 3, $5.5 in year 4, and $6 in year 5. Afterwards, they will maintain a zero growth dividend policy. R=15%. Calculate the stock price. Classic Potato Chips is thinking of distributing a coupon on its 454 g bag of regular crinkled chips. Currently, the cost of producing a bag of chips is $0.60, with typical expenses of $0.21 per bag. Classic sells the bag to retailers for $1.50 per bag. The coupon is in the amount of $0.60, while retailers are offered $0.13 for handling expenses. Classic forecasts an increase in sales of 500,000 bags with the coupon, incurring total marketing expenses of $115,000. What is its profit (or loss) per unit if Classic goes forward with the coupon? For full marks your answer(s) should be rounded to the nearest cent. Profit/loss = $ 0.00 10. Consider a Trading desk is trying to hedge their risk by delta hedging and other similar methods. Assume you have a portfolio of one security and have two different options on that security in your trading book. The Delta Gamma and Vega of option 1 and 2 and that of the portfolio is given below: (18 points) 3. Haz un grfico con los datos de la tabla adjunta. Empieza en el ao 0 y termina en 2007. Responde:a. A qu tipo de crecimiento corresponde, lineal o exponencial?b. Qu bucle de retroalimentacin coincide con este crecimiento? Enumera algunos factores que hayan contribuido a este aumento progresivo de la poblacin mundial. Cardiorespiratory endurance is the body capacity to deliver ____ to the exercising tissues. a. carbon dioxide b. carbon monoxide c. glucose d. oxygen Jennifer buys a typewriter for $590. She puts 20% down and will pay $70 a month for the next 10 months. What's the total amount of monthly payments?$700 ---- correct answer$750$850$1,000 greg is paid for the number of hours he works or for the number of units he produces, whichever is higher. which form of compensation does greg receive? implementation of an enterprise resource planning (erp) system provides an organization with an opportunity to also upgrade the information technology that it uses. (True or False) all the other patients were called to see the dentist before mr. johnson. mr. johnson Consider the following example: Assume that Hilary Taylor from New Orleans, Louisiana borrows $2,500 for four years at 7% add-on interest to be repaid in 48 monthly installments. 3. How did Georgians feel about the British? How didthis impact the Revolution?Lesson ? consider a bond paying a coupon rate of 11.75% per year semiannually when the market interest rate is only 4.7% per half-year. the bond has two years until maturity. a. find the bond's price today and six months from now after the next coupon is paid.